40

$x,y,z \geqslant 0$, $x+y^2+z^3=1$, prove $$x^2y+y^2z+z^2x < \frac12$$

This inequality has been verified by Mathematica. $\frac12$ is not the best bound. I try to do AM-GM for this one but not yet success. The condition $x+y^2+z^3$ is very weird.

HN_NH
  • 4,361

8 Answers8

18

The standard way of solving the problem on a conditional extremum is the method of Lagrange multipliers, which reduces it to a system of equations.

The greatest value of function $$f(x,y,z,\lambda) = x^2y+y^2z+z^2x+\lambda(x+y^2+z^3-1)$$ on the interval $$x,y,z\in[0,1]$$ is reached or at its edges, or in the inner stationary point.

$\color{brown}{\textbf{Inner stationary points.}}$

The inner stationary points has zero partial derivatives $$\begin{cases} f'_\lambda = x + y^2 + z^3 - 1 = 0\\ f'_x = z^2 + 2xy + \lambda = 0\\ f'_y = x^2 + 2yz + 2\lambda y = 0\\ f'_z = y^2 + 2zx + 3\lambda z^2 = 0. \end{cases}$$ After the excluding of parameter $\lambda$ get the system $$\begin{cases} x + y^2 + z^3 - 1 = 0\\ x^2 + 2yz = 2y(z^2 + 2xy)\\ y^2 + 2zx = 3z^2(z^2 + 2xy), \end{cases}$$ or $$\begin{cases} x + y^2 + z^3 - 1 = 0\\ (1-3y^2-z^3)^2-4y^4+2yz(1-z)=0\\ 2z(1-3yz)(1-y^2-z^3)+y^2-3z^4=0. \end{cases}$$

Using of Groebner basis allows to get the positive solutions $$ \genfrac{[}{.}{0}{0}{x\approx 0.16367,\quad y\approx 0.761982,\quad z\approx 0.634724,\quad f\approx 0.454882} {x\approx 0.558113,\quad y\approx 0.498412,\quad z\approx 0.578371,\quad f\approx 0.485622}. $$ Alternative way is shown below.

$\color{brown}{\textbf{The edges.}}$

The edges of the field are achieved when $x = 0$, $y = 0$ or $z = 0$.

Substitution $x = 0$ in the expressions for the partial derivatives leads to the system $$ \begin{cases} x=0\\ y^2+z^3 = 1\\ 2yz+2\lambda y = 0\\ y^2+3\lambda z^2 = 0 \end{cases} $$ with solutions $$ \genfrac{[}{.}{0}{} {x=0,\quad y=\sqrt{0.75},\quad z=\sqrt[3]{0.25}\approx 0.629991,\quad f\approx 0.47247} {x=0,\quad y=0,\quad z=1,\quad f=0}$$

Substitution $y = 0$ in the expressions for the partial derivative leads to the system $$ \begin{cases} y=0\\ x+z^3=1\\ z^2+\lambda = 0\\ 2zx+3\lambda z^2 = 0 \end{cases} $$ with solution $$x=0.6,\quad y=0,\quad z=\sqrt[3]{0.4}\approx 0.736806,\quad f\approx 0.32573.$$

Substitution $z = 0$ in the expressions for the partial derivative leads to the system $$ \begin{cases} z=0\\ x+y^2=1\\ 2xy+\lambda = 0\\ x^2+2\lambda y = 0 \end{cases} $$ with solutions $$ \genfrac{[}{.}{0}{} {x=0.8,\quad y=\sqrt{0.2}\approx 0.447214,\quad z=0,\quad f\approx 0.286217} {x=0,\quad y=0,\quad z=1,\quad f=0} $$

The values of function at the vertices of the area (unit parallelepiped) equal to zero.

So the greatest value approximately equals to $0.485622$. Given accuracy of calculations provides the inequality $$\boxed{x^2y+y^2z+z^2x<1/2.}$$

$\color{brown}{\textbf{System resolving, alternative way.}}$

Set condition allows us to reduce the problem to finding unconditional extremes of $$f(y,z)=(1-y^2-z^3)^2y+y^2z+z^2(1-y^2-z^3).$$ Necessary optimality conditions in the field have the form: $$\begin{cases} f'_y = (1-y^2-z^3)^2-4y^2(1-y^2-z^3)+2yz-2yz^2 = 0\\ f'_z = -6yz^2(1-y^2-z^3)+y^2+2z(1-y^2-z^3)-3z^4 = 0, \end{cases}$$ or $$\begin{cases} 5y^4+y^2(6z^3-6)+y(2z-2z^2)+(z^3-1)^2 = 0\\ 6y^3z^2+y^2(1-2z)+6y(z^3-1)z^2-5z^4+2z = 0. \end{cases}$$

If to consider the coefficient of the highest power of $y$ as denominator in equation, and the remaining coefficients - numerators, we get the above equations. Then we can subtract the second equation factor $y$ from the first and repeat subtraction with factor $1$, obtaining the system

$$\begin{cases} C_{2,2}(z)y^2 + C_{2,1}(z)y + C_{2,0}(z) = 0\\ 6y^3z^2+y^2(1-2z)+6y(z^3-1)z^2-5z^4+2z = 0, \end{cases}$$ where $$C_{2,2}(z) = 36z^7-36z^4+20z^2-20z+5,$$ $$C_{2,1}(z) = 18z^6+102z^5-30z^2,$$ $$C_{2,0}(z) = 36z^{10}-72z^7+50z^5+11z^4-20z^2+10z.$$

Thus, the order of the first equation in $y$ reduced from fourth to second. Likewise, lowering the order of the second equation by a first equation, we obtain

$$\begin{cases} C_{2,2}(z)y^2 + C_{2,1}(z)y + C_{2,0}(z) = 0\\ D_{2,2}(z)y^2 + D_{2,1}(z)y + D_{2,0}(z) = 0,\\ \end{cases}\qquad(1)$$ where $$D_{2,2}(z) = 180z^8+576z^7-72z^5-144z^4+40z^3-60z^2+30z-5,$$ $$D_{2,1}(z) = 180z^7-30z^6-30z^5-60z^3+30z^2,$$ $$D_{2,0}(z) = 180z^{11}-252z^8+100z^6-28z^5+25z^4-40z^3+40z^2-10z.$$

The system $(1)$ is a linear in the unknowns $y^2$ and $y$, so $$y^2=\dfrac{\Delta_2(z)}{\Delta_0(z)},\quad y=\dfrac{\Delta_1(z)}{\Delta_0(z)},\qquad(2)$$ where $$\Delta_0(z) = C_{2,2}(z)D_{2,1}(z) - C_{2,1}(z)D_{2,2}(z),$$ $$\Delta_2(z) = -C_{2,0}(z)D_{2,1}(z) + C_{2,1}(z)D_{2,0}(z),$$ $$\Delta_1(z) = -C_{2,2}(z)D_{2,0}(z) + C_{2,0}(z)D_{2,2}(z),$$ or $$\Delta_0(z) = 90z^{10}-828z^9-1662z^8-144z^7+396z^6+1028z^5-200z^4+180z^3-220z^2+10z,$$ $$\Delta_2(z) = -90z^{13}+540z^{12}+30z^{11}+234z^{10}-864z^9-290z^8+256z^7+74z^6+220z^5-160z^4+100z^3-50z^2,$$ $$\Delta_1(z) = 576z^{13}-1296z^{10}+90z^9+815z^8+444z^7-5z^6-580z^5+126z^4+10z^3+80z^2-30z-5.$$

From $(2)$ for $\Delta_0(z)\not=0$ should be $$\Delta_1^2(z) - \Delta_2(z)\Delta_0(z) = 0,$$ or $$331776z^{26}-1484892z^{23}-19440z^{22}+1233720z^{21}+3079404z^{20}+195732z^{19}-3189924z^{18}-3109428z^{17}+368233z^{16}+2734116z^{15}+1243978z^{14}-741000z^{13}-805907z^{12}+75696z^{11}+164040z^{10}+82560z^9-172194z^8+53440z^7-10290z^6+32440z^5-7460z^4-4400z^3+100z^2+300z+25 = 0.$$

The coefficients are particially calculated using the Mathcad package, and $\mathcal{polyroots}()$ function is also used, which calculates all the roots of the polynomial by the "accompanying matrix" method.

Calculating values $y$ with $(2)$ and $x,f$ by the formula $$x = 1-y^2-z^3,\quad f=xy^2+yz^2+zx^2$$ and checking them by substituting in the original system, we obtain the following stationary points with the positive coordinates: $$ (x,y,z,f)\in\left[\genfrac{}{}{0pt}{0} {(0.5581125,\ 0.4984120,\ 0.5783713,\ 0.4856221)} {(0.1636702,\ 0.7619816,\ 0.6347238,\ 0.4548812)} \right. $$

4

Update: A simpler prof (2023/03/24)

Since $y^2 + \frac{1}{4} \ge y$, it suffices to prove that $$x^2(y^2 + \tfrac{1}{4}) + y^2 z + z^2 x < \frac{1}{2}.$$ Since $y^2 = 1-x-z^3$, it suffices to prove that $$4x^3+(4z^3-5)x^2+(-4z^2+4z)x+4z^4-4z+2 > 0. \tag{1}$$

We have \begin{align*} &4x^3+(4z^3-5)x^2+(-4z^2+4z)x+4z^4-4z+2\\ =\ & 4x\left(x + \frac{1}{2}z^3 - \frac{5}{8}\right)^2 + \left( 4\,z-4\,{z}^{2}-{z}^{6}+\frac52\,{z}^{3}-{\frac {25}{16}} \right) x+4\,{z}^{4}-4\,z+2. \tag{2} \end{align*}

It suffices to prove that $$\left( 4\,z-4\,{z}^{2}-{z}^{6}+\frac52\,{z}^{3}-{\frac {25}{16}} \right) x+4\,{z}^{4}-4\,z+2 > 0.$$

Since $4z^4 - 4z + 2 = (2z^2 - 1)^2 + (2z - 1)^2 > 0$, we only need to prove the case that $4\,z-4\,{z}^{2}-{z}^{6}+\frac52\,{z}^{3}-{\frac {25}{16}} < 0$. Since $x + z^3 \le 1$, it suffices to prove that $$\left( 4\,z-4\,{z}^{2}-{z}^{6}+\frac52\,{z}^{3}-{\frac {25}{16}} \right) (1 - z^3) + 4\,{z}^{4}-4\,z+2 > 0$$ or $$16\,{z}^{9}-56\,{z}^{6}+64\,{z}^{5}+65\,{z}^{3}-64\,{z}^{2}+7 > 0.$$

We have \begin{align*} &16\,{z}^{9}-56\,{z}^{6}+64\,{z}^{5}+65\,{z}^{3}-64\,{z}^{2}+7\\ ={}& \frac{1}{16} ( 16{z}^{4}-1 ) ^{2}z+ \frac{7}{2}( 4{z}^{2}-1 ) ^{2} ( 1-z ) z+ \frac58( 4{z}^{2}-1 ) ^ {2}z + 7 ( 2z-1 ) ^{2} ( 1-z ) z\\[6pt] &\qquad + 42(z - 9/16)^2 z + \frac{87}{4} z^2 - \frac{3133}{128}z + 7\\ >{}& 0 \end{align*} where we use $\frac{87}{4} z^2 - \frac{3133}{128}z + 7 > 0$ for all $z \in [0, 1]$.

We are done.

River Li
  • 37,323
3

This answer is incomplete.

Let $A=x+y^2$, $B=y^2+z^3$ and $C=z^3+x$.

Claim: $x^2y \leq \dfrac{A}{2}x^{3/2}$, $y^2z \leq \dfrac{B^{2/3}}{4^{2/3}}y^{4/3}$, $z^2x \leq \dfrac{C^{4/3}}{4^{2/3}}x^{1/3}$.

Assuming the claim, we note that $A+B+C=2$ and we need to prove that the given sum is less than $\dfrac{A+B+C}{4}$.

Aravind
  • 6,150
3

enter image description here

The method to be employed here is exactly the same as in a previous answer:

Let $u = x$ , $v = y^2$ , $w = z^3$ , then $u,v,w \ge 0$ , $u+v+w=1$ and the inequality to be established: $$ x^2y+y^2z+z^2x < \frac12 \quad \Longrightarrow \quad f(u,v,w) = u^2v^{1/2}+v\,w^{1/3}+w^{2/3}u < \frac12 $$ The maximum of this function inside the equilateral triangle must shown to be less than $1/2$.
There is no symmetry argument, because the latter is effectively destroyed by the "weird" condition $x+y^2+z^3=1$ , as it is called. Another proof without words is attempted by plotting a contour map of the function, as depicted. Levels (nivo) of these isolines are defined (in Delphi Pascal) as:

nivo := min + sqrt(g/grens)*(max-min); { sqrt = square root ; grens = 25 ; g = 0..grens }
The darkness of the isolines is proportional to the (positive) function values; they are almost black near the maximum and almost white near the minimum values. Maximum and minimum values of the function are observed to be:

 4.58251457205350E-0003 < f < 4.85621276951755E-0001 < 1/2
The little $\color{blue}{\mbox{blue}}$ spot is where $\left|f(u,v,w) - \mbox{max}\right|< 0.0002$ . This maximum is close to values found by other people here, but not quite. Perhaps it's interesting to know the position of the maximum as well:

(x,y,z) = ( 5.58304528246164E-0001 , 4.97693736095187E-0001 , 5.78892473099889E-0001 )
I have no idea how to convert these numerical values into something more analytical.

EDIT. What you see is what you get :-) Without doubt. Some decent error analysis reveals that the value of the maximum to be trusted in this answer is : $\;0.48562 \pm 0.00003\;$ , quite in agreement with values given elsewhere (e.g. in the comment by Steve Kass).

Han de Bruijn
  • 17,070
2

I don't think the best bound can be solved for analytically.

using Lagrange multipliers , I tried to maximize $x^2y+y^2z+z^2x$ $\quad$ subject to the constraint $x+y^2+z^3=1$

maximize $g=x^2y+y^2z+z^2x+\lambda(x+y^2+z^3-1)$

$\frac {\partial g}{\partial x}=0:2xy+z^2+\lambda=0$

$\frac {\partial g}{\partial y}=0:x^2+2yz+2\lambda y=0$

$\frac {\partial g}{\partial z}=0:y^2+2zx+3\lambda z^2=0$

$\frac {\partial g}{\partial \lambda}=0:x+y^2+z^3-1=0$

$x\frac {\partial g}{\partial x}=0:2x^2y+z^2x+\lambda x=0$

$y\frac {\partial g}{\partial y}=0:x^2y+2y^2z+2\lambda y^2=0$

$z\frac {\partial g}{\partial z}=0:y^2z+2z^2x+3\lambda z^3=0$

$3(x^2y+y^2z+z^2x)=-\lambda (x+2y^2+3z^3)=(2xy+z^2)(x+2y^2+3z^3)$

If the global maximum occur on $ x, y, z \ge 0 $ then we can solve $(2xy+z^2)(x+2y^2+3z^3) \ge 0 $ for $x, y, z \ge 0 $

If no local maximum occur at $ x, y,z \ge 0 $ then we have to search for farthest points from the point of global minimum.

ibnAbu
  • 1,966
  • It looks like all we have to do is search for the farthest point away from the local minimum that also satisfies the constraint. – ibnAbu May 24 '16 at 07:58
2

Let $x=0$.

Then $y^2+z^3=1 \Rightarrow z^3=1-y^2$

Seek to maximise $p=y^2z=y^2(1-y^2)^{\frac 13}$

WLOG let $u=y^2$ so that $p=u(1-u)^{\frac 13}$

$\frac {dp}{du}=(1-u)^{\frac 13}-u{\frac 13}(1-u)^{-\frac 23}$

$\frac {dp}{du}=\frac 13(3-4u)(1-u)^{-\frac 23}$

Maximum is at $u=\frac 34$

Thus maximum is $p=\frac 34(\frac 14)^{\frac 13}$

$p \le \frac 34(\frac 14)^{\frac 13}$

$p^3 \le \frac {27}{64}\frac 14$

$p^3 \le \frac {27}{256} < \frac {32}{256}$

$p^3 < \frac {1}{8}$

$p < \frac {1}{2}$

We then have to demonstrate that any increase in $x$ creates corresponding decreases in $y^2$ and $z^2$ such that the other two terms in the expression $x^2y+y^2z+z^2x$ increase more slowly than the decrease in $p$.

tomi
  • 9,594
  • 3
    You find a maximum at $p = \frac{3}{4}\left(\frac{1}{4}\right)^{\frac{1}{3}} \approx 0.47247$. However, in the comments to the question, Ewan Delanoy found a point which has value $0.484818$. So I don't think your last paragraph will work? – sTertooy May 24 '16 at 12:07
  • 1
    I was hoping someone else would address that... – tomi May 24 '16 at 12:12
  • It also seems odd that the maximum I found is at $\frac {\sqrt 3} 2$. Is there some trig transformation that would make this easier? – tomi May 24 '16 at 12:14
  • @stalker2133 look there: http://math.stackexchange.com/questions/1775498/x-y-z-geqslant-0-xy2z3-1-prove-x2yy2zz2x-frac12/1801749#1801749 – Yuri Negometyanov May 27 '16 at 18:26
  • @stalker2133 There are two my answers. Follow link, please. – Yuri Negometyanov May 27 '16 at 19:37
  • @stalker2133: Welcome in 2016! Computers exist for more than half a century now. Instead of "thinking hard" and accomplishing essentially nothing, wouldn't it be wise to simply use these machines for the purpose of exploring computationally feasible problems, like this sort of inequalities? Everybody who is employing mathematics in practice is doing this on a routinely base, except mathematicians themselves. Why? – Han de Bruijn May 28 '16 at 19:12
1

$\color{green}{\textbf{Computer free version of 17.12.20.}}$

$\color{brown}{\textbf{Preliminary notes.}}$

We should prove the inequality$\;f(x,y,z)= x^2y+y^2z+z^2x \le \frac12\;$ under the conditions $x+y^2+z^3=1,\;$ $x,y,z\in [0,1].$

In accordance with the @RiverLi answer,


  • $y\le y^2+\frac14,\tag1$
  • $2-4(x^2y+y^2z+z^2x)\ge 2-x^2-(4x^2+4z)(1-x-z^3)-4z^2x,\tag2$

where $(1)$ follows from the evident $\;\left(y-\frac12\right)^2 \ge0\;$.

At the same time, for $\;t\in[0,1]\;$ $$t^4-t+\frac3{4\sqrt[\large3]4} = \left(t^2-\sqrt[\large3]2t-\frac1{2\sqrt[\large3]2}\right)\left(t^2 +\sqrt[\large3]2 t +\frac3{2\sqrt[\large3]2}\right) $$ $$= \left(t-\frac1{\sqrt[\large3]4}\right)^2\left(t^2+\sqrt[\large3]2 t +\frac3{2\sqrt[\large3]2}\right) \ge 0,$$

$$t\le t^4+H,\quad \text{where}\quad H=\frac3{4\sqrt[\large3]4} = \dfrac12\sqrt[\large3]{\dfrac{27}{32}}<\dfrac12,\quad H\approx0.47247.\tag3$$

$\color{brown}{\textbf{The edges of area.}}$

Taking in account $(1)$ and $(3),$ one can get \begin{align} &f(x,y,0) = x^2y \le (1-y^2)^2y\,\bigg|_{\large y=\frac1{\sqrt5}} = \dfrac{16}{25\sqrt5}<0.28622, \\[4pt] &f(x,0,z) = z^2x = (1-z^3)z^2 \le (1-z^3)z \le H, \\[4pt] &f(0,y,z) = y^2z = (1-z^3)z \le H. \end{align}

Thus, the given inequality is correct on the edges of the area.

$\color{brown}{\textbf{The inner stationary points.}}$

Taking in account $(2)$ and $(1)$ in the form of $\;4z\le 4z^2+1,\;$ it suffices to prove the inequality $\;g(x,z)\ge0,\;$ where $$g(x,z) = 2-x^2-(4x^2+1+4z^2)(1-x-z^3)-4z^2x,$$ $$g(x,z) = 1+4x^2z^3 + (4z^5+z^3-4z^2) + (4x^3-5x^2+x),\tag4$$ under the conditions $\;\;x,z\in(0,1).$

Stationary points should satisfy the system $\;g'_x = g'_z =0,\;$ or \begin{cases} g'_x = 8z^3x + 12 x^2 - 10x + 1 = 0\\[4pt] g'_z = 12x^2z^2+20 z^4+3z^2-8z = 0,\tag5 \end{cases}

and then \begin{cases} 12x^2z = 8-3z-20z^3\hspace{163mu} \left[z^{-1}\times(5.2)\right]\\ 8z^4x +(8-3z-20z^3) - 10xz + z = 0,\qquad \left[z\times(5.1)\right]\\ \end{cases} $$x = h_1(z) = \dfrac{4-z-10z^3}{5z-4z^4},\quad x^2 = h_2(z) = \dfrac{8-3z-20z^3}{12z}\tag6.$$

At the same time, from $(4),(5)$ should \begin{align} &5g(x,z) = 5g - xg'_x - z g'_z\\[4pt] & = 5+20x^2z^3 + (20z^5+5z^3-20z^2) + (20x^3-25x^2+5x)\\[4pt] & - x(8z^3x + 12 x^2 - 10x + 1) - z(12x^2z^2+20 z^4+3z^2-8z)\\[4pt] & = 5 + 4 x + 8x^3 -15x^2 - 12 z^2 + 2 z^3, \end{align} $$5g(x,z) = 5 - 15h_2(z) + 4 h_1(z) (1+2h_2(z)) - 12 z^2 + 2 z^3.\tag7$$ Substitution of $(6)$ to $(7)$ represents the inequality $\;g(x,z) \ge 0\;$ in the form of $$256 - 568 z + 501 z^2 - 1280 z^3 + 1180 z^4 - 300 z^5 + 1600 z^6 - 624 z^7 - 96 z^8\ge 0.\tag8$$

Since $$h(z)-1=\dfrac{4z^4-10z^3-6z+4}{5z-4z^4}=\dfrac{2(1-2z)(2+z+2z^2-z^3)}{5z-4z^4},$$ then $\;h(z)>1,\;$ if $\;z\in\left(0,\frac12\right).\;$

Besides, $\;h(z)\;$ has a positive denominator and decreasing numerator in the interval $\;z\in(0,1],\;$ wherein $\;h\left(\frac7{10}\right) = -\frac{325}{6349}<0.\;$ Therefore, $\;h(z)< 0,\;$ if $\;z\in\left[\frac7{10},1\right].\;$

Thus, all inner stationary points belong to the interval $\;z\in\left[\dfrac12,\dfrac7{10}\right).\;$

And now, substitution $\;z=\dfrac{6+t}{10},\;t\in[-1,1),\;$ transforms $(8)$ to the evident inequality $$-3 t^8 - 339 t^7 - 6214 t^6 - 13083 t^5 + 1041140 t^4\\ + 12923432 t^3 + 64494426 t^2 - 898904 t + 13278632 \ge 0.$$

Done!

  • 1
  • Should it be $f(x,0,z) = z^2x = (1-z^3)z^2 \le (1-z^3)z \le H,
  • f(0,y,z) = y^2z = (1-z^3)z \le H$?

    – River Li Dec 18 '20 at 03:32
  • 1
  • How do you get the 3rd equation in (5)? How do you get the equation between (5) and (6)? More details should be given.
  • – River Li Dec 18 '20 at 03:32
  • 1
  • Also, the 1st equation in (4) is different from RHS of (2)?
  • – River Li Dec 18 '20 at 03:41